1. Trang chủ
  2. » Luận Văn - Báo Cáo

(Luận văn) bất đẳng thức muirhead và một số vấn đề liên quan

51 4 0

Đang tải... (xem toàn văn)

Tài liệu hạn chế xem trước, để xem đầy đủ mời bạn chọn Tải xuống

THÔNG TIN TÀI LIỆU

Nội dung

ĐẠI HỌC THÁI NGUYÊN TRƯỜNG ĐẠI HỌC KHOA HỌC BÙI VIỆT LONG lu an BẤT ĐẲNG THỨC MUIRHEAD VÀ MỘT SỐ VẤN ĐỀ LIÊN QUAN n va p ie gh tn to w d oa nl LUẬN VĂN THẠC SĨ TOÁN HỌC nf va an lu z at nh oi lm ul z m co l gm @ an Lu THÁI NGUYÊN - 2016 n va ac th si ĐẠI HỌC THÁI NGUYÊN TRƯỜNG ĐẠI HỌC KHOA HỌC BÙI VIỆT LONG lu BẤT ĐẲNG THỨC MUIRHEAD VÀ MỘT SỐ VẤN ĐỀ LIÊN QUAN an n va p ie gh tn to oa nl w LUẬN VĂN THẠC SĨ TOÁN HỌC d Chuyên ngành: Mã số: PHƯƠNG PHÁP TOÁN SƠ CẤP 60 46 01 13 nf va an lu lm ul z at nh oi NGƯỜI HƯỚNG DẪN KHOA HỌC PGS.TS HÀ TRẦN PHƯƠNG z m co l gm @ an Lu THÁI NGUYÊN - 2016 n va ac th si i Mục lục Mở đầu Chương Bất đẳng thức Muirhead lu an 1.1.1 Một số khái niệm 1.1.2 Định lý Muirhead hai ba số 1.1.3 Một số ví dụ n va 1.1 Bất đẳng thức Muirhead cho trường hợp hai ba số ie gh tn to p 1.2 Bất đẳng thức Muirhead tổng quát 11 nl w 1.2.1 Định lý Muirhead trường hợp n biến 11 d oa 1.2.2 Bất đẳng thức Muirhead mở rộng 15 lu 23 nf va an Chương Một số áp dụng bất đẳng thức Muirhead 2.1 Chứng minh số bất đẳng thức đại số hình học 23 lm ul 2.1.1 Một số bất đẳng thức đại số 23 z at nh oi 2.1.2 Một số bất đẳng thức hình học 36 2.2 Kết hợp với số bất đẳng thức khác 40 z 2.2.1 Một số bất đẳng thức liên quan 40 @ 47 m co Kết luận l gm 2.2.2 Ví dụ áp dụng 42 48 an Lu Tài liệu tham khảo n va ac th si MỞ ĐẦU Bất đẳng thức vấn đề nghiên cứu hình thành từ sớm toán học sơ cấp thu hút quan tâm lu nhiều tác giả Đây phần kiến thức đẹp đẽ, thú vị tốn sơ cấp Do vấn đề bất đẳng thức hút nhiều người an n va nghiên cứu toán sơ cấp có nhiều tập sử dụng để thi kỳ thi học sinh giỏi quốc gia quốc tế Đã có nhiều tác giả ngồi nước tn to có nghiên cứu bất đẳng thức có nhiều chuyên đề hay, thể ie gh tính thời vấn đề nghiên cứu p Được hình thành vào đầu kỷ XX, bất đẳng thức Muirhead xuất cơng trình nghiên cứu nhà toán học R F Muirhead nl w d oa vào năm 1903 tổng quát hóa quan trọng bất đẳng thức AM − GM Nó cho đánh giá tổng Symmetric hai số có lu nf va an quan hệ ≺ Có thể nói, bất đẳng thức Muirhead cơng cụ mạnh việc giải số tốn bất đẳng thức có độ phức tạp cao thể lm ul việc có nhiều tập thi học sinh giỏi, Olympic nước, z at nh oi khu vực, giới - mà việc giải cần dùng đến bất đẳng thức Muirhead Hơn nữa, bất đẳng thức Muirhead áp dụng với bất đẳng z thức khác để xây dựng bất đẳng thức sâu sắc Mặc dầu có nhiều tác giả quan tâm đến bất đẳng thức Muirhead việc cải l gm @ tiến bất đẳng thức chậm, kỷ sau (năm 2009) kể từ cơng trình R F Muirhead, hai tác giả J B Paris A Vencovská m co đưa cải tiến bất đẳng thức an Lu Sự lựa chọn đề tài Bất đẳng thức Muirhead số vấn đề liên quan nhằm giới thiệu lại cơng trình nghiên cứu R F Muirhead n va J B Paris A Vencovská đánh giá tổng Symmetric hai số ac th si thực khơng âm có quan hệ ≺ Ngoài luận văn giới thiệu số ví dụ áp dụng bất đẳng thức Muirhead việc chứng minh tập bất đẳng thức sử dụng kỳ thi học sinh giỏi, Olympic nước, khu vực, giới Luận văn chia thành hai chương Chương nhằm giới thiệu kiến thức lý thuyết bất đẳng thức Muirhead mở rộng bất đẳng thức Trong Chương chúng tơi giới thiệu ví dụ toán sử dụng đến bất đẳng thức Muirhead áp dụng định lý Muirhead lu Luận văn thực hoàn thành trường Đại học Khoa học - Đại học Thái Nguyên Qua xin chân thành cảm ơn thầy cô an n va tn to giáo Khoa Toán, Ban Giám hiệu, Phòng Đào tạo nhà trường Quý Thầy Cơ giảng dạy lớp Thạc sĩ khóa (6/2014- 6/2016) trường Đại học ie gh Khoa học - Đại học Thái Nguyên tận tình truyền đạt kiến thức quý báu, trang bị kiến thức tạo điều kiện tốt cho p trình học tập nghiên cứu d oa nl w Tơi xin bày tỏ lịng biết ơn chân thành tới PGS TS Hà Trần Phương, người tận tình bảo, tạo điều kiện giúp đỡ tơi có thêm nhiều kiến cách hoàn chỉnh nf va an lu thức, khả nghiên cứu, tổng hợp tài liệu để hoàn thành luận văn z at nh oi lm ul Tôi xin gửi lời cảm ơn đến gia đình, bạn bè đồng nghiệp động viên, giúp đỡ tơi q trình học tập Do thời gian trình độ cịn hạn chế nên luận văn khơng tránh khỏi z thiếu sót Tơi mong nhận góp ý thầy bạn để luận văn hồn thiện l gm @ Tôi xin chân thành cảm ơn! Thái Nguyên, tháng năm 2016 co m Người viết luận văn an Lu va Bùi Việt Long n ac th si Chương Bất đẳng thức Muirhead Bất đẳng thức Muirhead cho trường hợp hai ba số 1.1.1 Một số khái niệm lu 1.1 an n va tn to Định nghĩa 1.1 ([6]) Cho n số thực không âm a = (a1 , a2 , , an ) gh số thực dương x = (x1 , x2 , , xn ) Ta định nghĩa p ie i) Tổng Cyclic (Viết tắt: cyc) xa11 xa22 xann đại lượng X a a x11 x22 xann =xa11 xa22 xann + xa21 xa32 xa1n nl w cyc oa d n + · · · + xan1 xa12 xan−1 an lu nf va ii) Tổng Symmetric (Viết tắt: sym) xa11 xa22 xann đại lượng X a a X a an 2 n T (a) = T (x; a) = x1 x2 xn = xσ(1) xaσ(2) xaσ(n) , lm ul sym σ∈S(n) z at nh oi tổng sym lấy tất hoán vị (σ(1), σ(2), , σ(n)) (1, 2, , n), S(n) tập hợp tất hoán vị {1, 2, , n} z iii) Trung bình Symmetric xa11 xa22 xann đại lượng @ T (x; a) n! co l gm [x; a] = an Lu cho T (x; a) phần tử x xác định rõ m Ta sử dụng kí hiệu ngắn gọn [a] thay cho kí hiệu [x; a], T (a) thay n va ac th si Ví dụ 1.1 ([2]) X ab2 c3 = ab2 c3 + bc2 a3 + ca2 b3 ; cyc X abc = 6abc sym Ví dụ 1.2 ([4]) Với a = (1, 3, 2) x = (x1 , x2 , x3 ) T (x; a) = x1 x32 x23 + x1 x33 x22 + x2 x31 x23 + x2 x33 x21 + x3 x31 x22 + x3 x32 x21 Và lu [x; a] = (x1 x32 x23 + x1 x22 x33 + x2 x31 x23 + x2 x33 x21 + x3 x31 x22 + x3 x32 x21 ) an va Ví dụ 1.3 ([6]) n n ie gh tn to (n − 1)! 1X [(1, 0, 0, , 0); (x1 , , xn )] = (x1 + x2 + + xn ) = xi n! n i=1 p trung bình cộng số x1 , , xn   √ 1 ( ; ; ); (x1 , , xn ) = n x1 x2 xn n n n oa nl w d trung bình nhân số x1 , , xn an lu Mệnh đề 1.1 ([6]) nf va Nếu x1 x2 xn = lm ul z at nh oi [a1 , a2 , , an ] = [(a1 − r), (a2 − r), , (an − r)] với r > cho − r > z Nếu x1 x2 xn > @ với r > cho − r > co l gm [a1 , a2 , , an ] > [(a1 − r), (a2 − r), , (an − r)] m Sử dụng bất đẳng thức AM – GM, với hai số thực không âm a b ta có   [a] + [b] a+b > 2 an Lu n va ac th si Nhận xét 1.1 Cho số thực không âm a = (a1 , a2 , , an ) số thực dương x = (x1 , x2 , , xn ) Nếu b = (aσ(1) , aσ(2) , , aσ(n) ), (σ(1), σ(2), , σ(n)) hoán vị {1, 2, , n} ta ln có T (x; a) = T (x; b), [x; a] = [x; b] Tiếp theo ta giới thiệu số khái niệm so sánh n số Cho n số thực không âm a = (a1 , a2 , , an ) Dễ thấy ta ln xếp lại trật tự phần tử a để cho a1 > a2 > · · · > an lu an Do luận văn này, khơng tính tổng qt ta ln giả n va tn to thiết a1 > a2 > · · · > an nói đến n số (a) Ta xem xét khái niệm quan hệ ≺ hai n số thông qua định nghĩa sau gh Định nghĩa 1.2 ([6]) Cho hai n số thực không âm a = (a1 , a2 , , an ) p ie b = (b1 , b2 , , bn ) Ta nói b trội a, kí hiệu a ≺ b hay b  a điều kiện sau thỏa mãn (sau xếp lại trật tự phần tử w oa nl a, b cần thiết): d 1) a1 > a2 > · · · > an ; b1 > b2 > · · · > bn ; lu nf va an 2) a1 + a2 + · · · + am b1 + b2 + · · · + bm với m : m n − 1; 3) a1 + a2 + · · · + an = b1 + b2 + · · · + bn z at nh oi lm ul Ví dụ 1.4 ([4]) (2, 1, 0) ≺ (3, 0, 0); (0, 2, 1) ≺ (0, 0, 3), (4, 0, 0, 0) 6≺ (2, 0, 2) số phần tử hai khác nhau, z (5, 0, −1) 6≺ (2, 2, 0) có phần tử âm bộ, @ (4, 1, 1, 1) 6≺ (3, 3, 1, 0)vì + 6> + m co l gm (2, 1, 1, 1) 6≺ (1, 1, 1, 1)vì + + + 6= + + + 1, Ví dụ 1.5 ([6]) n n va n an Lu 1 1 , , , ≺ (1, 0, , 0) | {z } n n n | {z } ac th si 1.1.2 Định lý Muirhead hai ba số Định lý 1.2 (Định lý Muirhead hai số, [2]) Cho số thực dương a1 , a2 , b1 , b2 thỏa mãn:    a1 > a2 ; b > b ; a1 > b1 ;   a1 + a2 = b1 + b2 Cho x, y số thực dương, X X xb1 y b2 xa1 y a2 > sym sym lu an Đẳng thức xảy : a1 = b1 , a2 = b2 x = y n va p ie gh tn to Định lý 1.3 (Định lý Muirhead cho ba số, [2]) Cho hai ba số thực dương a1 , a2 , a3 , b1 , b2 , b3 thỏa mãn:    a1 > a2 > a3 ; b1 > b2 > b3 ; a1 > b1 ; a1 + a2 > b1 + b2 ;   a1 + a2 + a3 = b1 + b2 + b3 oa nl w d Cho x, y, z số thực dương, X X xa1 y a2 z a3 > xb1 y b2 z b3 an lu sym nf va sym z at nh oi lm ul Đẳng thức xảy : = bi ; i = 1, 2, x = y = z Chứng minh Để chứng minh định lý ta cần đến bổ đề sau: Bổ đề 1.4 ([1]) Cho số thực không âm a1 , a2 , b1 , b2 , thỏa mãn: a1 + z a2 = b1 +b2 ; max {a1 ; a2 } > max {b1 ; b2 } Khi với số thực dương x, y , ta có: xa1 y a2 + xa2 y a1 > xb1 y b2 + xb2 y b1 l gm @ m co Đẳng thức xảy a1 = b1 ; a2 = b2 x = y n va a1 > a2 , a1 > b1 , b1 > b2 an Lu Chứng minh Khơng tính tổng qt, ta giả sử ac th si Do a1 + a2 = b1 + b2 nên ta có: xa1 y a2 + xa2 y a1 − xb1 y b2 − xb2 y b1 = xa2 y a2 (xa1 −a2 + y a1 −a2 − xb1 −a2 y b2 −a2 − xb2 −a2 y b1 −a2 ) = xa2 y a2 (xb1 −a2 + y b1 −a2 )(xb2 −a2 − y b2 −a2 ) = a a (xb1 + y b1 )(xb2 − y b2 ) > x 2y Bổ đề chứng minh Ta tiếp tục chứng minh định lý Ta xét hai trường hợp sau: lu i) Trường hợp Nếu b1 > a2 , điều kéo theo a1 > a1 + a2 − b1 từ an a1 > b1 ta có va n a1 > max {a1 + a2 − b1 , b1 } tn to Kéo theo Từ p ie gh max {a1 , a2 } = a1 > max {a1 + a2 − b1 , b1 } oa nl w a1 + a2 − b1 > b1 + a3 − b1 = a3 d a1 + a2 − b1 > b2 > b3 an lu nf va ta có max{a1 + a2 − b1 , a3 } > max{b2 , b3 } lm ul sym z at nh oi Áp dụng Bổ đề 1.4 hai lần ta có: X X xa1 y a2 z a3 = z a3 (xa1 y a2 +xa2 y a1 ) cyc = X > X = X z a3 (xa1 +a2 −b1 y b1 +xb1 y a1 +a2 −b1 ) z > X gm @ cyc xb1 (y a1 +a2 −b1 z a3 +y a3 z a1 +a2 −b1 ) n va sym xb1 y b2 z b3 an Lu cyc m xb1 (y b2 z b3 +y b3 z b2 ) co l cyc ac th si 34 Ví dụ 2.12 ([8]) Nếu a, b, c là số thực, không âm ta có bất đẳng thức sau a3 + b3 + c3 + abc ≥ (a + b + c)3 Chứng minh Thật vậy, bất đẳng thức dễ dàng có (a + b + c)3 = [3, 0, 0] + 18 [2, 1, 0] + 36 [1, 1, 1] Sau phải chứng minh lu an n va 3[3, 0, 0] + 6[1, 1, 1] ≥ (3[3, 0, 0] + 18[2, 1, 0] + 36[1, 1, 1]) gh tn to Khi đó, p ie   18 36 18 [3, 0, 0] + − [1, 1, 1] ≥ [2, 1, 0] 7 nl w Hay d oa   36 18 ([3, 0, 0] − [2, 1, 0]) + − [1, 1, 1] ≥ 7 nf va an lu Hiển nhiên: (3, 0, 0)  (2, 1, 0) , lm ul suy z at nh oi [3, 0, 0] ≥ [2, 1, 0] z  gm @ [1, 1, 1] ≥ Nhận xét 2.3 Thấy Định lý Muirhead đặc biệt hiệu lớp l co toán mà dấu đẳng thức xẩy tất biến Thực m tiễn có trường hợp ngoại lệ.Ta xét trường hợp biến x, y, z điều kiện kèm phải a1 , a2 , a3 , b1 , b2 , b3 > 0; an Lu n va ac th si 35 Khi bất đẳng thức Muirhead xẩy = bi ,∀i = 1, x = y = z có hai số nhau; số cịn lại số x, y, z Ví dụ 2.13 ([1]) Cho x, y, z ≥ 0, thỏa mãn xy + yz + zx = Chứng minh rằng: 1 + + ≥ x+y y+z z+x Chứng minh Bất đẳng thức cần chứng minh tương đương với bất đẳng lu an thức sau: n va  (xy + yz + zx) 2 ≥ 25 tn to 1 + + x+y y+z z+x gh Hay ie !2 p X 4(xy + yz + zx) ≥ 25(x + y)2 (y + z)2 (z + x)2 (x + y)(x + z) nl w cyc d oa Hay lu xy X an x2 + X cyc xy ≥ 25 X x2 y + 2xyz sym cyc nf va cyc !2 !2 ! X lm ul sym ! x3 y z + 38x2 y z ≥ l gm sym sym @ sym x4 yz + 14 X sym z + X sym z at nh oi Rút gọn bất đẳng thức ta bất đẳng thức tương đương sau đây: ! ! X X X X x5 y − x4 y + x5 y − x3 y Bất đẳng thức cuối theo Muirhead co m Đẳng thức xảy xy + yz + zx = 1và x = y; z = an Lu n va y = z; x = x = z; y = hay x = y = 1; z = y = z = 1; x = x = z = 1; y =  ac th si 36 2.1.2 Một số bất đẳng thức hình học Ví dụ 2.14 [IMO, 1961] Cho a, b, c độ dài ba cạnh tam giác ABC , S diện tích tam giác Khi √ 3S a2 + b2 + c2 Chứng minh Cách Sử dụng cơng thức Heron, ta viết lại bất đẳng thức cho sau r √ (a + b + c) (a + b − c) (a + c − b) (b + c − a) a2 + b2 + c2 > 2 2 lu Bình phương hai vế bất đẳng thức, ta an n va a4 + b4 + c4 > a2 b2 + b2 c2 + c2 a2 gh tn to Ta dùng ký hiệu [a], bất đẳng thức tương đương với p ie [(4, 0, 0)] > [(2, 2, 0)] w Hiển nhiên bất đẳng thức áp dụng bất đẳng thức Muir- oa nl head ứng với hai số (4, 0, 0)  (2; 2; 0) d Cách Đặt x = a + b − c; y = c + a − b; z = b + c − a, ta thu x + y + z = a + b + c Khi , Sử dụng cơng thức Heron ta có s q (a + b + c)2 (x + y + z)3 √ 4S = (a + b + c) (xyz) (a + b + c) = 27 3 nf va an lu lm ul z at nh oi Lúc ta cần chứng minh  (a + b + c)2 a2 + b2 + c2 z gm @ Bất đẳng thức cuối suy từ bất đẳng thức Muirhead vì: [(1, 1, 0)] ≺ [(2, 0, 0)] l an Lu [(1, 1, 0)] ≤ [(2, 0, 0)] m co hay nên bất đẳng thức cuối Điều suy bất đẳng thức cần chứng n va minh  ac th si 37 Ví dụ 2.15 ([3]) Xét tam giác ABC Với độ dài cạnh a, b, c R, r bán kính đường trịn ngoại tiếp, nội tiếp tam giác Khi r R r h ih ih i 2 2 2 2a − (b − c) 2b − (c − a) 2c − (a − b) (a + b) (b + c) (c + a) (2.7) Chứng minh Trước hết ta bất đẳng thức phải chứng minh với biến x, y, z cách dùng đồng thức S a+b+c abc ; r = ; S = p (p − a) (p − b) (p − c) ; p = R= 4S p lu đặt an n va a= y+z z+x x+y ;b = ;a = 2 tn to Khi đó, ta có ie gh x = b + c − a > 0; y = c + a − b > 0; z = a + b − c > p Bình phương hai vế Bất đẳng thức (2.7) rút gọn ta được: nl w 105[(4; 4; 4)] + 264[(5; 4; 3)] + 88[(6; 3; 3)] + 48[(7; 3; 2)] + 9[(8; 2; 2)] d oa 136[(5; 5; 2)] + 106[(6; 4; 2)] + 176[(6; 5; 1)] + 7[(6; 6; 0)] an lu + 72[(7; 4; 1)] + 8[(7; 5; 0)] + 8[(8; 3; 1)] + [(8; 4; 0)] nf va Sử dụng bất đẳng thức Muirhead, ta có 48[(7; 3; 2)] 48[(7; 4; 1)], 88[(6; 3; 3)] 88[(6; 5; 1)], z at nh oi lm ul 9[(8; 2; 2)] 8[(8; 3; 1)] + [(8; 4; 0)], 264[(5; 4; 3)] 136[(5; 5; 2)] + 106[(6; 4; 2)] + 22[(6; 5; 1)], z gm @ 105[(4; 4; 4)] 66[(6; 5; 1)] + 7[(6; 6; 0)] + 24[(7; 4; 1)] + 8[(7; 5; 0)] Cộng vế tương ứng bất đẳng thức trên, ta bất đẳng thức l m co cần chứng minh Dấu đẳng thức xảy x = y = z , tức tam giác ABC  an Lu Ví dụ 2.16 ([6]) Cho a, b, c số thực dương, n va (a + b) (b + c) (c + a) > 8abc ac th si 38 Chứng minh Cách 1: Khai triển rút gọn ta bất đẳng thức tương đương a2 b + a2 c + b2 c + b2 a + c2 b > 6abc Như vậy, bất đẳng thức cần chứng minh tương đương cần chứng minh tương với X a2 b > 6abc sym Vì (2, 1, 0)  (1, 1, 1) nên theo bất đẳng thức Muirhead ta có [2, 1, 0] ≥ [1, 1, 1] lu an Dấu đẳng thức xảy tam giác ABC va n Cách 2: p ie gh tn to Áp dụng bất đẳng thức AM − GM với số thực dương  √  a + b > ab  √ b + c > bc  √  c + a > ab oa nl w d Suy (a + b)(b + c)(c + a) > 8abc Điều kéo theo bất đẳng thức cần lu nf va an chứng minh Dấu đẳng thức xảy a = b = c (Dấu đẳng thức xảy tam giác ABC đều.)  z at nh oi lm ul Ví dụ 2.17 ([8]) Nếu a, b, c độ dài ba cạnh tam giác bất kỳ, chứng minh a2 + b2 b2 + c2 c2 + a2 a3 b3 c3 a+b+c≤ + + ≤ + + 2c 2a 2b bc ca ab z l gm @ Chứng minh Quy đồng, biến đổi khử mẫu ta bất đẳng thức cho tương đương với: m co 2(a2 bc+ab2 c+abc2 ) ≤ ab(a2 +b2 )+bc(b2 +c2 )+ca(c2 +a2 ) ≤ 2(a4 +b4 +c4 ), an Lu Tương đương với: n va [2, 1, 1] ≤ [3, 1, 0] ≤ [4, 0, 0] ac th si 39 Đến ta sử dụng định lý Muirhed, ta có (2, 1, 1) ≺ (3, 1, 0) ≺ (4, 0, 0) Bất đẳng thức cho theo định lý Muirhead Đẳng thức xảy a = b = c, hay tam giác  Ví dụ 2.18 ([2]) Cho a,b,c độ dài cạnh tam giác ABC Chứng minh b c a+b+c a + + ≤ a2 + 2bc b2 + 2ca c2 + 2ab ab + bc + ca lu an n va p ie gh tn to Chứng minh Vì a,b,c độ dài cạnh tam giác ABC nên a,b,c>0; ! X X X 1X a2 bc + a3 b + a3 bc ab sym sym sym sym ! X X X X ≤ a a2 b c + a3 b3 + a4 bc sym sym sym sym d oa nl w Điều kéo theo X X X 1X 3 2 a b c+ a b c +2 a b +2 a4 b c sym sym sym sym X X X 9X 2 +2 a3 b c + a4 b2 c + a3 b2 c2 ≤ abc sym sym sym sym X X X X +2 a4 b3 + a3 b3 c + a5 bc + a4 b2 c, nf va an lu lm ul sym sym X a5 bc + X 3X 2 3X 3 a b c ≥2 a4 b c + a b c sym sym sym z sym @ gm Khi X X 3X 2 3X a bc ≥ a3 b + ab sym sym sym m co sym a4 + l sym z at nh oi hay sym an Lu n va Theo định lý Schur định lý Muirhead ta có: 3 [4; 0; 0] + [2; 1; 1] ≥ [3; 1] ( Theo định lý Schur) 2 ac th si 40 1 (4; 0; 0)  (2; 2; 0) (Theo định lý Muirhead) 2 (3; 1; 0)  (2; 2; 0) (Theo định lý Muirhead) Suy hạng tử bất đẳng thức cuối không âm (đpcm.) Đẳng thức xẩy a = b = c Hay tam giác ABC  2.2 Kết hợp với số bất đẳng thức khác Trong phần giới thiệu số bất đẳng thức mà việc giải cần kết hợp bất đẳng thức Muirhead với bất thức khác: bất đẳng lu thức Schur, ASYM, Cauchy-Schwarz Trước hết ta nhắc bất đẳng thức an n va Một số bất đẳng thức liên quan gh tn to 2.2.1 Bất đẳng thức Cauchy-Schwarz ([2]) p ie w Với hai n số thực (a1 , a2 , , an ), (b1 , b2 , , bn ) ta ln có: d oa nl n n n X X X 2 ( b i ) ( )( b2i ) i=1 i=1 i=1 an lu Đẳng thức xảy khi: nf va lm ul a2 an a1 = = ··· = b1 b2 bn Bất đẳng thức Holder ([2]) z at nh oi với qui ước bk = ak = z @ gm Dạng tổng quát: j=1 j=1 i=1 m i=1 co l v X m m X n n uY Y um m t ( aij ) > aij an Lu aij > với i = 1, 2, , m, j = 1, 2, , n n va ac th si 41 Bất đẳng thức ASYM ([2]) Kết hợp bất đẳng thức AM – GM Holder, ta có bất đẳng thức ASYM sau: Cho m số n số thực không ấm a1 = (a11 ; a12 ; a1n ), (a21 ; a22 ; a2n ), , am = (am1 ; am2 ; amn ) Ta đặt a1i + a2i + · · · + ami m với i = 1, , n kí hiệu t = (t1 , , tn ) Khi với mơi số dương x = (x1, x2 , , xn ) ta có ti = lu m X [x; ] > m[x; t] an n va i=1 gh tn to Bất đẳng thức Schur số hệ p ie Ta nhắc lại với số thực không âm a = (a, a2 , , an ) số thực dương x = (x1 , x2 , , xn ), tổng sym xác định sau: X a n xσ(1) xaσ(2) xaσ(n) , T (x; a) = oa nl w d σ∈S(n) lu an tổng sym lấy qua tất hoán vị (σ(1), σ(2), , σ(n)) nf va {1, 2, , n}, S(n) tập hợp tất hoán vị {1, 2, , n} lm ul Định lý 2.1 [Bất đẳng thức Schur] Với α ∈ R β > 0, ta có z at nh oi T (x; (α + 2β, 0, 0)) + T (x; (α, β, β)) > T (x; (α + β, β, 0)) Một trường hợp đặc biệt bất đẳng thức Schur β = định lý z gm @ sau Định lý 2.2 ([2]) Cho x, y, z số thực không âm t ∈ R Khi co l ta có bất đẳng thức m xt (x − y)(x − z) + y t (y − x)(y − z) + z t (z − x)(z − y) > 0, an Lu đẳng thức xẩy x = y = z x = y, z = (và hoán n va vị nó) ac th si 42 Hệ 2.3 ([2]) Cho x, y, z a, b, c số thực dương thỏa mãn a b c a > b > c Khi ta có bất đẳng thức a(x − y)(x − z) + b(y − x)(y − z) + c(z − x)(z − y) > Hệ 2.4 ([2]) Cho x, y, z số thực dương Khi 3xyz + x3 + y + z > 2((xy)3/2 + (yz)3/2 + (zx)3/2 ) Hệ 2.5 ([2]) Cho k ∈ (0; 3] a, b, c số thực dương Khi (3 − k) + k(abc)2/3 + a2 + b2 + c2 > 2(ab + bc + ca) lu 2.2.2 Ví dụ áp dụng an n va Ví dụ 2.19 ([2]) Cho a, b, c ≥ Chứng minh rằng: gh tn to (a2 + b2 )(b2 + c2 )(c2 + a2 )(ab + bc + ca)2 ≥ 8a2 b2 c2 (a2 + b2 + c2 ) p ie Chứng minh Bất đẳng thức cho tương đương với: ! ! X X X X 1 a4 b2 + a2 b c a2 b2 + a2 bc sym sym sym sym ! 4X 2 1X X 2 abc a + ab ≥ sym sym sym d oa nl w nf va an lu Kéo theo a6 b + X a6 b2 c2 + sym a6 b c + X z at nh oi sym +2 X sym sym sym a5 b c + X a5 b c sym a4 b c ≥ sym X a6 b c + X gm sym a4 b4 c2 sym @ a4 b4 c2 + X X z + X ! lm ul X sym +2 sym X sym a6 b c + sym X sym a4 b4 c2 (2.8) n va sym a4 b c ≥ sym X an Lu sym m co l Bất đẳng thức tương đương với X X X X a6 b + a6 b c + a5 b c + a5 b c a4 b c ac th si 43 Theo định lý Muirhead, ta có: (6; 4; 0)  (6; 2; 2) ; (6; 3; 1)  (6; 2; 2) (2.9) Theo ASYM, ta có: 2[5; 4; 1] + 2[3; 5; 2] + 2[4; 3; 3] ≥ 6[4; 4; 2] (2.10) Từ (2.9), (2.10) suy (2.8) suy bất đẳng thức cho chứng minh Đẳng thức xảy a = b; c = b = c; a = a = c; b =  lu Ví dụ 2.20 ([2]) Cho a, b, c > 0; abc = Chứng minh   bc ca ab + + a4 + b4 + c4 + ≥ c a b an va n Chứng minh Đặt x = √ a; y = √ b; z = √ c suy xyz = Khi đó: gh tn to Bất đẳng thức cho tương đương với: p ie  x12 + y 12 + z 12 + 3x4 y z ≥ x6 y + y z + z x6 w Hay d oa nl X x12 + sym X x4 y z ≥ sym X x6 y sym lu nf va an Theo định lý Schur ta có [12; 0; 0] + [4; 4; 4] ≥ [6; 6; 0] lm ul z at nh oi Theo định lý Muirhead ta có (8; 4; 0)  (6; 6; 0) Suy hạng tử bất đẳng thức cuối khơng âm ta có điều phải chứng minh z Đẳng thức xẩy a = b = c = gm @  an Lu cyc m k > Chứng minh : X ak (a − b)(a − c) ≥ co l Ví dụ 2.21 [Định lý Schur] Cho a, b, c số thực không âm, cho n va ac th si 44 Chứng minh Vì bất đẳng thức đối xứng theo biến; khơng tính tổng qt, ta giả sử a ≥ b ≥ c ≥ đó, bất đẳng thức cho viết lại ak (a − b)(a − c) + bk (b − c)(b − a) + ck (c − a)(c − b) ≥ Bất đẳng thức theo định lý Muirhead, ta có: [k + 2; 0; 0] + [k; 1; 1]  [k + 1; 1; 0]  (Mọi hạng tử vế trái khơng âm.) lu Ví dụ 2.22 ([1]) Cho a, b, c > thỏa mãn abc = Khi √ √ √ a3 + b3 + c3 > a b + c + b a + c + c a + b an n va p ie gh tn to Chứng minh Thật vậy, bất đẳng thức cần chứng minh tương đương với !2 r r r √ √ √ abc abc abc (a3 + b3 + c3 ) > a b+c+b a+c+c a+b 2 w oa nl Theo bất đẳng thức Cauchy-Schwarz để chứng minh toán ta cần chứng minh khẳng định sau: d  lu 3 (b + c) + b abc nf va an (a + b + c ) > a abc X cyc hay a6 + X a3 b > z at nh oi lm ul Tương đương với (a + c) + c X abc a3 b2 c, sym sym ! ! a3 b − sym X a3 b c > sym gm sym a3 b c + X @ sym a6 − X z X  (a + b) m co l Bất đẳng thức cuối theo bất đẳng thức Muirhead Đẳng thức √ xảy a = b = c =  an Lu Ví dụ 2.23 ([2]) Cho a, b, c > Chứng minh    2 a2 + b2 b2 + c2 c2 + a2 (a + b + c)2 > a2 b2 + b2 c2 + c2 a2 n va ac th si 45 Chứng minh Chuẩn hóa a + b + c = 1, bất đẳng thức cần chứng minh trở thành : (a2 + b2 )(b2 + c2 )(c2 + a2 ) > 8(a2 b2 + b2 c2 + c2 a2 ) , tương đương với X X X X X X a3 b c a4 b2 c2 + a4 b3 c+ a5 b2 c+2 a5 b3 +2 a6 b + sym >3 sym sym X X a4 b4 +6 sym sym sym a4 b c sym sym Chuyển sang viết dạng [a], bất đẳng thức tương đương với lu an [6; 2; 0]+2[5; 3; 0]+2[5; 2; 1]+2[4; 3; 1]+[4; 2; 2]+[3; 3; 2] > 3[4; 4; 0]+6[4; 2; 2] va n Theo Định lý Muirhead, ta có tn to gh [6; 2; 0] > [4; 4; 0]; p ie 2[5; 3; 0] > [4; 4; 0] nl w Theo Bất đẳng thức ASYM, 5+4+3 2+1+3 1+3+2 ; ; ] 3 = 6[4; 2; 2] d oa 2[5; 2; 1] + 2[4; 1; 3] + 2[3; 3; 2] > 6[ nf va an lu z at nh oi lm ul Cộng vế theo vế đánh giá trên, ta có điều phải chứng minh Dấu 1 đẳng thức xảy khi: a = b = c = a = b = ; c = hốn vị  Ví dụ 2.24 [Iran, 1996] Cho x, y, z số thực dương Chứng minh rằng: 1 + + ) > (x + y)2 (y + z)2 (z + x)2 z l gm @ (xy + yz + zx)( m co Chứng minh Bất đẳng thức cho tương đương với: X X X X X x5 y + x4 yz + 6x2 y z − x4 y − x3 y − x3 y z > cyc sym cyc an Lu sym sym n va ac th si 46 Ta viết lại X X X X 5 x3 y ) x y− x y ) + 3( x y− ( sym sym sym sym + 2xyz(3x2 y z + X x3 − X x2 y) > sym cyc Theo bất đẳng thức Muirhead bất đẳng thức Schur, tổng ba số hạng không âm nên bất đẳng thức chứng minh Ví dụ 2.25 ([10]) Cho a, b, c > số thực, a3 + b + c + lu an > (a + b + c)3 27 n va ((2a + b)3 + (2b + a)3 + (2c + b)3 + (2a + c)3 + (2c + a)3 ) 81 p ie gh tn to Chứng minh Thật vậy, bất đẳng thức cần chứng minh tương đương với X X 3 3 (28(a +b +c )+3 a b+6abc) > (18(a +b +c )+18 a2 b) 27 3.27 sym sym w hay oa nl 16(a3 + b3 + c3 ) + 6abc > X a2 b sym d sym sym lm ul sym nf va an lu Tương đương với X X X 7( a3 − a2 b) + 2(a3 + b3 + c3 + 3abc − a2 b) > Từ bất đẳng thức Muirhead bất đẳng thức Schur, ta suy bất đẳng z at nh oi thức cuối  Chú ý Ví dụ áp dụng bất đẳng thức Karamata để chứng minh z m co l gm @ an Lu n va ac th si 47 KẾT LUẬN Trong luận văn chúng tơi trình bày số vấn đề sau: Giới thiệu số kết cổ điển bất đẳng thức Muirhead cho hai ba số thực không âm lu an Phát biểu chứng minh định lý Muirhead tổng quát cho n số thực không âm n va tn to Giới thiệu mở rộng bất đẳng thức Muirhead J B Paris ie gh A Vencovská chứng minh năm 2009 p Tổng hợp tập bất đẳng thức mà chứng minh sử dụng bất đẳng thức Muirhead kết hợp với số bất đẳng thức quen thuộc khác d oa nl w nf va an lu z at nh oi lm ul z m co l gm @ an Lu n va ac th si 48 Tài liệu tham khảo Tiếng Việt [1] Định lý Muihead ứng dụng, http://www.diendantoanhoc.net lu [2] Trần Phương, (2011), Những viên kim cương bất đẳng thức toán an học, NXB tri thức n va [3] Cezar L., Tudorel L., (2006), Problem 11245, American Mathematical Monthly, Vol.113 p ie gh tn to Tiếng Anh w [4] Hardy G.H., Littlewood J.E., Polya G (1967), Inequalities, Cambridge oa nl University Press d [5] Ivan M., (2007), Classical Inequalities, Olympial Training Materials an lu nf va [6] Lau C.H., (2006), Muirhead’s Inequality, Mathematical Excalibur, Vol.11 lm ul [7] Paris J.B and Vencovská A., (2009), A Generalization of Muirhead’s z at nh oi inequality, Journal of Mathematical inequalities, Vol.3 [8] Radmila B.M., Jose A.G.O, Rogelio V.D., (2009), Inequalities, A z Mathematical Olympial Approach, Birkhauser @ gm [9] Stanley R., (2006), On The Computer Solution of Symmetric m co l Homogeneous Triangle Inequalities, Alliant Computer Systems Corporation Littleton, MA 01460 an Lu [10] Zoran K.D.D., Milivoje L., Ivan M., (2005), Inequalities of Karamata, Schur and Muirhead, and some applications, The Teaching of n va M athematics, Vol.8, pp 31-45 ac th si

Ngày đăng: 24/07/2023, 09:00

TÀI LIỆU CÙNG NGƯỜI DÙNG

TÀI LIỆU LIÊN QUAN

w